Wednesday, July 28, 2010

FINAL INSTALLMENT OF FORMAL METHODS TUTORIAL

With this segment, I conclude my discussion of Arrow's General Possibility Theorem. And I think this will also conclude this tutorial on the use and abuse of formal methods in political philosophy. I will be happy to respond to questions, if there are any, but I think enough is enough. Thank you all for staying with me on this, for pointing out errors, and for asking questions. It has been fun for me, revisiting material I have not taught for twenty years or more, and I hope it has been informative and fun for you.

An extremely interesting result concerning the consistency of majority rule was produced by the Australian political scientist Duncan Black. In a book called The Theory of Committees and Elections, published in 1958, Black proved an important theorem about circumstances under which majority rule is guaranteed to produce a transitive social preference ordering. In a moment, I am going to go through the proof in detail, but let me first explain intuitively what Black proved. Ever since the French Revolution, political commentators have adopted a convention derived from the seating arrangement in the National Assembly. In that body, Representatives belonging to each party were seated together, and the groups were arrayed in the meeting hall in such a manner that the most radical party, the Jacobins, sat on the extreme left of the hall, and the most reactionary party, the Monarchists, sat on the right, with the other groups seated between them from left to right according to the degree to which their policies deviated from one extreme or the other. Thus was born the left-right political spectrum with which we are all familiar. [Of course, in the U. S. Senate, there are no Communists and only one Socialist, but, as the reign of George W. Bush shows, there are still plenty of Monarchists.]

The interesting fact, crucial for Black's proof, is that wherever a party locates itself on the spectrum, it tends to prefer the positions of the other parties, either to the left or to the right, less and less the farther away they are seated. So, if an individual identifies himself with a party in the middle, he will prefer that party's positions to those of a party a little bit to the left, and he will prefer the policies of the party a little bit to the left over those of a party farther to the left, and so on. The same is true looking to the right. Notice that since only ordinal preference is assumed, you cannot ask, "Is a party somewhat to the left of you farther from you than a party somewhat to the right of you?" [Make sure you understand why this is true. Ask me if it is not.]

Consider contemporary American politics. If I am a moderate Republican [assuming there still is one], I will prefer my position to that of a conservative Republican, and I will prefer that position to a right wing nut. I will also prefer my position to that of a Blue Dog Democrat [looking to my left rather than to my right], and that position to the position of a Liberal Democrat, and that position in turn to the position of a Socialist [Bernie Sanders?].

This can be summarized very nicely on a graph, along the X-axis of which you lay out the left-right political spectrum, while on the Y-axis you represent the order of your preference. Pretty obviously, the graph you draw will have a single peak -- namely, where your first choice is on the X-axis -- and will fall away on each side, going monotonically lower the farther you get on the X-axis from your location on it. In short, your preference, when graphed in this manner, will be single-peaked. Here is an example of a person's preference order graphed in this manner. For purposes of this example, there are five alternatives, (a, b, c, d, e), and the individual has the following preference order: d > e > c > b > a





Let us suppose that there is a second person whose preference order is a > b > c > d > e. It is obvious that if we posted this person's preferences on the same graph, the two together would look like this:



Notice that each of these lines has a single peak. The first individual's line peaks at alternative D; the second's at alternative A. If you do a little experimenting, you will find that if you change the order in which the alternatives are laid out on the X-axis, sometimes both lines are still single peaked, sometimes one remains single peaked and one no longer is. Sometimes neither is single peaked. Fr example, if you change the order slightly so that the alternatives are laid out on the X-axis in the order a B E D C, the first individual's line will still be single peaked, but the second individual's line will now be in the shape of a V with one peak at A and another peak at C. [Try it and see. It is too much trouble for me to draw it and scan it and size it and insert it.]

Suppose now that we have an entire voting population, each with his or her own preference order, and that we plot all of those preference orders on a single graph, a separate line for each person. There might be some way of arranging the alternatives along the X-axis so that everyone's preference order, when plotted on that graph, is single peaked. Then again, there might not be. For example, if you have three people and three alternatives, and if those three people have the preferences that give rise to the Paradox of Majority Rule, then there is no way of arranging the three alternatives along the X-axis so that all three individuals' preferences orders can be plotted on that graph single-peakedly. [Try it and see. Remember that mirror images are equivalent for these purposes, so there are really three possible ways of arranging the alternatives along the X-axis, namely xyz, xzy, and yxz.]

Duncan Black proved that if there is some way of arranging the available alternatives along the X-axis so that everyone's preference order, plotted on that graph, is single peaked, then majority rule is guaranteed to produce a consistent social preference order. Notice, in particular, that if everyone's preferences can be mapped onto the familiar left-right spectrum, with each individual preferring an alternative less and less the farther away it is in either direction from the most preferred alternative, then everyone will on that graph have a single peaked order [because it will peak at the most preferred alternative and fall away monotonically to the right and to the left.]

The proof is fairly simple. It goes like this.

Step 1: Assume that there are an odd number of individuals [the proof works for an even number of individuals, but in that case there can be ties, which produces social indifference, which then requires an extra couple of steps in the proof, so I am trying to make this as simple as possible.] Assume that their preferences can be plotted onto a graph so that all of the plots are single-peaked.

Step 2: Starting at the left, count peaks [there may be many peaks at the same point, of course, showing that all of those people ranked that alternative as first] and keep counting until you reach one more than half of the total number of peaks, i.e. (n/2 + 1). Assume there are p peaks to the left of that point, q peaks at that point, and r peaks to the right, with (p+q+r) = n. Now, by construction, (p+q) > n/2 and pn/2, because if (q+r)n/2, which by construction it is not.

Step 3: Let us call the alternative with the q peaks alternative x. Clearly, there is a majority of individuals who prefer x to every alternative to the right of x on the graph, because there are p+q individuals whose plots are downward sloping from x as you go to the right, which means they prefer x to everything to the right, and p+q is a majority. But there are q+r individuals who prefer x to everything to the left of x, because their plots are downward sloping as you go to the left, and q+r are a majority. So alternative x is preferred in a pairwise comparison by a majority to every other alternative.

Step 4: Remove alternative x from the graph, remove alternative x from everyone's preference order, and then redraw all of the plots. They will all still be single-peaked. Why? Well, there are three possible cases: Either the dot representing the individual's ranking of x was the peak, or it was to the left of the peak, or it was to the right. In each case, when you reconnect the remaining dots, the graph remains single-peaked [try it and see. It is too hard to draw it and scan it and upload it. But it is intuitively obvious.]

Step 5: You now have a new set of single-peaked plots on a single graph, so go through Steps 2 and 3 all over again. The winning alternative is preferred to every other remaining alternative, and is of course inferior to the first winner. If you now iterate this process until you run out of alternatives, you are left with a fully transitive social preference established by repeated uses of majority rule.

Black's theorem has considerable real world application, as we have seen, but it of course does not identify necessary and sufficient conditions for majority rule to produce a transitive social preference order. It only identifies a sufficient condition, namely single-peakedness. This means that there are sets of individual preferences that cannot be mapped single-peakedly onto a single graph, and yet which by majority rule produce transitive social preference orders. I leave it to you to construct an example of this.

Monday, July 26, 2010

COLLECTIVE C HOICE FIFTH INSTALLMENT

Proof of Arrow's Theorem

Step 1.     By Condition P, there is at least one decisive set for each ordered pair, namely the set of all the individuals. From all the decisive sets, choose a smallest decisive set, V, and let it be decisive for some ordered pair (x,y). What I mean is this: Consider each set of individuals that is decisive for some ordered pair or other. Since there is a finite number of individuals, each of these sets must have some finite number of individuals in it. And the sets may have very different numbers of individuals in them. But one or more of them must be the smallest set. So arbitrarily choose one of the smallest, call it V, and label the pair of alternatives over which it is decisive (x,y).

Step 2:     By Condition P, V cannot be empty. [Go back and look at Condition P and make sure you see why this is so. It is not hard]. Furthermore, by Lemma 3, V cannot have only one member [because Lemma 3 proved that no single individual, i, can be decisive for any ordered pair (x,y) ]. Therefore, V must have at least two members.

Step 3: Partition the individuals 1, 2, ......, n in the following way:

The set of all individuals

---------------------------------------------------------------

| |

V V3

---------------------

V1 V2

    Where V1 = a set containing exactly one individual in V

        V2 = the set of all members of V except the one individual in V1

        V3 = the rest of the individuals, if there are any.

    Is this clear? V is a smallest decisive set. It must have at least two individuals in it. So it can be divided into V1 containing just one individual, and V2 containing the rest of V. V3 is then everyone else, if there is anyone else not in the smallest decisive set V.

Step 4: Now let the individuals in the society have the following rankings of three alternatives, x, y, and z. [And now you will see how this is an extension of the original Paradox of Majority Rule with which we began.]

    V1: x > y and y > z

    V2: z> x and x> y

    V3: y>z and z>x

[You see? This is one of those circular sets of preference orders: xyz, zxy, yzx]

    V1 is non-empty, by construction.

    V2 is non-empty, by the previous argument.

    V3 may be empty.

Step 5: a) By hypothesis, V is decisive for x against y. But V is the union of V1 and V2, and xPiy for all i in V1 and V2. Therefore, xPy. [i.e., the society prefers x to y.]

    b) For all i in the union of V1 and V3, yPiz. For all j in V2, zPjy. If zPy, then V2 is decisive for (x,y). But by construction, V2 is too small to be decisive for anything against anything, because V2 is one individual smaller than a smallest decisive set, V. Therefore not zPy. Hence, yRz [see the definitions of P and R].

    c) Therefore xPz by Lemma 1(f) [go back and look at it].

    d) But xP1z and zPix for all i not in V1, so it cannot be that xPz, because that would make V1 decisive for (x,z), which contradicts Lemma 3. Therefore, not xPz.

Step 6: The conclusion of Step 5d) contradicts Step 5c). Thus, we have derived a contradiction from the assumption that there is a Social Welfare Function that satisfies Conditions 1', 3, P, and 5. Therefore, there is no SWF that satisfies the four Conditions. Quod erat demonstrandum.

    OK. Everybody, take a deep breath. This is a lot to absorb. Arrow's Theorem is a major result, and it deserves to be studied carefully. Go back and re-read what I have written and make sure you understand every step. It is not obscure. It is just a little complicated. If you have questions, post them as a comment to this blog and I will answer them.

Friday, July 23, 2010

COLLECTIVE CHOICE FOURTH INSTALLMENT

    [end of Arrow third installment]

Proof of Lemma 3:        Assume xDy for i [i,e,, i is decisive for x against y]

    The proof now proceeds in two stages. First, for an environment [x,y,z], constructed by adding some randomly chosen third element z to x and y, we show that i is a dictator over [x,y, z].

    Then we show how to extend this result step by step to the conclusion that i is a dictator over the entire environment S of admissible alternatives.

First Stage: Proof that i is a dictator over the environment [x,y,z]

(step i)    Construct a set of individual orderings over [x,y,z] as follows.

    Ri: x > y > z [i.e., individual i's ordering of the three]

    All the other Rj: yPjx yPjz Rj[x,z] unspecified

    In other words, we will prove something that is true regardless of how everyone other than i ranks x against z.

(step ii)    xPiy by construction. But, by hypothesis xDy for i. Therefore xPy

    In words, i is assumed to strongly prefer x to y, and since by hypothesis i is decisive for x against y, the society also strongly prefers x to y.

(step iii)     For all i, yPiz, by construction Therefore, yPz, by Condition P, and xPz by Lemma 1(c). In words, since everyone strongly prefers y to z, so does the society. And since the society strongly prefers x to y and y to z, it strongly prefers x to z [since Axiom II, which is used to prove Lemma 1(c), stipulates that the SWF is transitive.]

(step iv) So xPz when xPiz, regardless of how anyone else ranks x and z. [check the construction of the individual orderings in step (i) ]

(step v)    Hence xḎz for i, which is to say that i dictates over the ordered pair (x,z)

(step vi)    Now consider (y,z) and assume the following set of individual orderings:

    Ri:    y > x > z

    All the other Rj: yPjx zPjx and Rj[y,z] unspecified.

(step vii)    yPix for all i. Therefore yPx by Condition P

(step viii)    xḎz for i, by (v). Hence xPz.

(step ix)    So yPz by Lemma 1(c). Thus yḎz for i.

    In words, we have now shown that i dictates over the ordered pair (y,z). Let us take a minute to review what is going on here. We are trying to prove that if i is decisive for a single ordered pair, (x,y), then i is a dictator over an environment consisting of x, y, and some randomly chosen z. If we can show that i is a dictator for every ordered pair in the environment [x,y,z] then we shall have shown that i is a dictator over that environment. There are six ordered pairs that can be selected from the environment, namely (x,y), (x,z), (y,x), (y,z), (z,x), and (z,y). So we must establish that i dictates over every single one of these ordered pairs. We have already established that i dictates over (x,z) in step (v) and over (y,z) in step (ix).

(step x) We can now extend this argument to the other four ordered pairs that can be selected from the environment [x,y,z]. In particular, let us do this for the ordered pair (y,x). Construct the following set of orderings:

    Ri:    y > z > x

    All the other Rj: zPjy zPjx Rj[x,y] unspecified.

(step xi)      zPix for all i. Hence zPx by Condition P

(step xii)    yḎz for i by (step ix). Hence yPz

(step xiii)    So yPx by Lemma 1(c). Thus yḎx for i.

    So we have proved [or can do so, by just iterating these steps a few more times] that i dictates over every ordered pair in [x,y,z], and therefore i is a dictator over the environment [x,y,z]. So much for Stage One of the proof of Lemma 3. Now, take a deep breath, review what has just happened to make sure you understand it, and we will continue to:

Stage Two: The extension of our result to the entire environment, S, of available alternatives. Keep in mind that S, however large it may be, is finite.

    Assume xDy for i [our initial assumption -- just repeating for clarity] and also assume the result of Stage One. Now consider any ordered pair of alternatives (z,w) selected from the environment S. There are just seven possibilities.

    1. x = z     w is a third alternative

    2. x = w     z is a third alternative

    3. y = z     w is a third alternative

    4. y = w     z is a third alternative

    5. x = z     y = w

    6. y = z    x = w

    7. Neither z nor w is either x or y

Case 1:        We have an environment consisting of three alternatives: [x=z, y, w]. Stage One shows that if xDy for i, then x=zḎw for i.

Case 2, 3, 4:    Similarly

Case 5:        Trivial

Case 6:        Add any other element v to form the environment [x=w, y=z, v]. From x=wDy=z for i, it follows that y=zḎx=w for i. [In words, just in case you are getting lost: In the case in which y is element z and x is element w, from the fact that i is decisive for x against y, which is to say for w against z, , it follows that i dictates over y and x, which is to say over z and w. This is just a recap of Stage One.

case 7: This is the only potentially problematic one case, and it needs a little explaining. We are starting from the assumption that i is decisive for x against y, and we want to show that i is a dictator over some totally different of alternatives z and w, so we are going to creep up on that conclusion, as it were. First we will add one of those two other alternatives, z, to the two alternatives x and y to form the environment [x,y,z]. From Stage One, if xDy for i then xḎz for i. But trivially, since xḎz for i, it follows that xDz for i. [The point is that if i dictates over x and z, then of course i is decisive for x against z].

    Now add w to x and z to form the environment [x,z,w]. Since xDz for i, it follows that zḎw for i, by Stage One. In words, if i is decisive for x against z, then in the environment [x,z,w], i dictates over z and w. This follows from Stage One. What this shows is just how powerful Lemma 3 really is.

    Thus we have demonstrated that xDy for i implies zḎw for i, for all z and w in S. In other words, if i is decisive for some ordered pair (x,y), then i is a dictator over S. But Condition 5 stipulates that no individual may be a dictator. Therefore:

    An acceptable Social Welfare Function does not permit any individual to be decisive for even a single ordered pair of alternatives in the environment S of available alternatives.

    Can we all say Ta-Da? This is the heavy lifting in Arrow's theorem. Using this Lemma, we can now fairly quickly prove that there is no SWF satisfying Axioms I and II and all four Conditions, 1', 3, P, and D.

Wednesday, July 21, 2010

COLLECTIVE CHOICE THEORY THIRD INSTALLMENT

This is really a devastating theorem. Basically, it says that there is no voting mechanism that gets around the Paradox of Majority Rule. The proof proceeds as follows. First Arrow states a set of little results about the relations R, I, and P. You are already familiar with them. They are trivial, as we shall see. Then he proves a little Lemma about the choice function. Then he proves a big important Lemma that is really the guts of the theorem. Finally, he uses the Lemmas to prove what is essentially an extension of the Paradox of Majority Rule, and he is done. We are going to go through this slowly and carefully. Let us start with the two little lemmas. Lemma 1 and Lemmas 2.

Lemma 1:    (a)    For all x, xRx

        (b)    If xPy then xRy

        (c)    If xPy and yPz then xPz

        (d)    If xIy and yIz then xIz

        (e)    For all x and y, either xRy or yPx

        (f)    If xPy and yRz then xPz

    These all follow immediately from the definitions of R, I, and P, the assumptions of transitivity and completeness, and truth functional logic. Arrow includes them as an omnibus Lemma because at one point or another in his proof he will appeal to one or another of them. You should work through all the little proofs as an exercise. I will go through just one to show you what they look like.

        (e) xRy or yRx [completeness]

         So if not xRy, then yRx.

         But the definition of yPx is yRx and not xRy

         Therefore, either xRy or yPx

Lemma 2:    xPy if and only if x is the sole element of C([x,y])

    If you review the definition of the Choice set, you will see that this Lemma is intuitively obvious. It says that in the little environment, S, consisting of nothing but x and y, if xPy, then x is the only element in the Choice set, C(S). Since this is a bi-conditional [if and only if], we have to prove it in each direction.

    a. Assume xPy. Then xRy, by Lemma 1(b). [See, this is why he put those little things in Lemma 1]. Furthermore, xRx, by Lemma 1(a). So x is in C([x,y]), because it is at least as good [i.e., R] as each of the elements of S, namely x and y. But if xPy then not yRx. Therefore, y is not in C([x,y]). So x is the sole element of C([x,y]).

    b. Assume x is the sole element of C([x,y]). Since y is not in C([x,y]), not yRx. Therefore xPy.

Lemma 3:    If an individual, i, is decisive for some ordered pair (x,y) then i is a dictator.

    This is a rather surprising and very important Lemma. It is the key to the proof of Arrow's theorem, and shows us just how powerful the apparently innocuous Four Conditions really are. To understand the Lemma, you must first know what is meant by an ordered pair and then you must be given three definitions, including one for the notion of "decisive."

    Easy stuff first. An ordered pair is a pair in a specified order. An ordered pair is indicated by curved parentheses. Thus, the ordered pair (x,y) is the pair [x,y] in the order first x then y. As we shall see, to say that individual is decisive for some ordered pair (x,y) is to say that i can, speaking informally, make the society choose x over y regardless of what anyone else thinks. But a person might be decisive for x over y and not be decisive for y over x. We shall see in a moment how all this works out. Now let us turn to the three definitions that Arrow is going to make use of in the proof of Lemma 3.

    Definition 1: "A set of individuals V is decisive for (x,y)" =df "if xPiy for all i in V and yPjx for all j not in V, then xPy"

    In other words, to say that a set of individuals V is decisive for the ordered pair (x,y) is to say that if everyone in V strongly prefers x to y, and everyone not in V strongly prefers y to x, then the society will strongly prefer x to y. Under majority rule, for example, any set of individuals V that has at least one more than half of all the individuals in the society in it is decisive for every ordered pair of alternatives (x,y).

    Definition 2: "xḎy for i" or "i dictates over (x,y)" =df "If xPiy then xPy"

    In words, we say that individual i dictates over the ordered pair (x,y) if whenever individual i strongly prefers x to y, so does the society regardless of how everyone else ranks x and y. [Notice that the capital letter D has a little line underneath it.]

    Definition 3: "xDy for i" or "i is decisive for (x,y)" =df "If xPiy, and for all j not equal to i, yPjx, then xPy."

    In words, i is said to be decisive for the ordered pair (x,y) if when i strongly prefers x to y and everyone else strongly prefers y to x, the society prefers x to y. [Notice that in this definition, the capital letter D does not have a little line underneath it.]

    Ok. Now we are ready to state and prove the crucial Lemma 3.

Lemma 3: If xDy for i, then zḎw for i, for all z,w in S

    In words, what this says is that if any individual, i, is decisive for some ordered pair (x,y) then that individual i is a dictator [i.e., dictates over any ordered pair (z,w) chosen from S]. This is an astonishing result. It says that if the Social Welfare Function allows someone to compel the society to follow her ranking of some ordered pair, no matter what, against the opposition of everyone else, then the Social Welfare Function makes her an absolute dictator. [L'ėtat c'est moi]. Here is the proof. It is going to take a while, so settle down. In order to make this manageable, I must use the various symbols we have defined. Let me review them here, so that I do not need to keep repeating myself.

    An ordered pair is indicated by curved parentheses: (x,y), as opposed to a non-ordered pair, which is indicated by brackets: [x,y].

    xḎy for i, which is D with a line under it, means "i dictates over (x,y)" (an ordered pair)

    xDy for i, which is D with no line under it, means "i is decisive for (x,y)"

Monday, July 19, 2010

COLLECTIVE CHOICE THEORY SECOND INSTALLMENT

From here on, I am going to break the exposition into short bits, because this is hard, and I do not want to lose anyone. My apologies to those of you who are having no trouble following it.    

    First of all, notice that Arrow assumes only ordinal preference. This means that there is no way in the proof to take account of intensity of preference, only order of preference. Let me give an example to make this clearer. In 1992, George H. W. Bush, Bill Clinton, and H. Ross Perot ran for the Presidency. There were some devoted followers of Perot who were crazy about him, and almost indifferent between Bush and Clinton, whom they viewed as both beltway politicians. Let us suppose that one of these supporters ranked Perot first, way ahead of the other two, and gave the edge slightly to Bush over Clinton, perhaps because Bush was a Republican. A second Perot supporter might have been rather unhappy with the choices offered that year, but preferred Perot slightly over Bush, while hating Clinton passionately. From Arrow's perspective which is that of ordinal preference, these two voters had identical preferences, namely Perot > Bush > Clinton, and an Arrovian SWF would treat the two individual preference orders as interchangeable.

    Now, there are many ways in which citizens in America can give expression to the intensity of their preferences, as political scientists are fond of pointing out. One is simply by bothering to vote. Voter enthusiasm, in a nation half of whose eligible voters routinely fail to go to the polls, is a major determinant of the outcome of elections. A second way is by contributing to campaigns, volunteering for campaign work, and so forth. Yet another way is through a vast array of voluntary organizations dedicated to pursuing some issue agenda or advantaging some economic or regional group. None of this can find expression in the sort of Social Welfare Function Arrow has defined. This is a very important limitation on the method of collective decision that we call voting. Now, there are voting schemes that allow voters to give expression to the intensity of their preferences [such as giving each voter a number of votes, which he or she can spread around among many candidates or concentrate entirely on one candidate], but these too are ruled out by Arrow, who only allows the SWF to take account of individual ordinal preferences.

    The second thing to note is that the requirement of completeness placed upon the SWF rules out partial orderings, such as those established by Pareto-Preference. It is often the case that every individual in the society prefers some alternative x to some other alternative y, and if there are a number of such cases, a robust partial ordering might be established that, while not complete, nevertheless allows the society to rank a sizeable number of the available alternatives. This option too is ruled out by Arrow's two axioms. These observations have the virtue of helping us to understand just how restricting a collective decision-making apparatus like majority rule is.

    We are now ready to state the four conditions that Arrow defines as somehow capturing the spirit of majoritarian democracy. Arrow's theorem will simply be the proposition that there is no Social Welfare Function, defined as he has in the materials above, that is compatible with all four conditions. In the original form of the proof, the conditions were, as you might expect, called Conditions 1, 2, 3, 4, and 5. In the revised version, which I shall be setting forth here, they are called Conditions 1' [a revised version of Condition 1], Condition 3 [which also is sometimes called the Independence of Irrelevant Alternatives], Condition P [for Pareto], and Condition 5. Here they are. I will tell you now that Condition 3 is the kinky one.

Condition 1': All logically possible rankings of the alternative social states are permitted. This is a really interesting condition. What it says, formally speaking, is that each individual may order the alternatives, x, y, z, ... in any consistent way. What it rules out, not so obviously, is any religious or cultural or other constraint on preference. For example, if among the alternatives are various dietary rules, or rules governing abortions, or rules governing dress, nothing is ruled in or ruled out. The individuals are free to rank alternatives in any consistent manner.

Condition 3: Let R1, R2, ......, Rn and R1', R2', .... Rn' be two sets of individual orderings of the entire set of alternatives x, y, z, .... and let C(S) and C'(S) be the corresponding social choice functions. If, for all individuals i and all alternatives x and y in a given environment S, xRiy if and only if xRi'y, then C(S) and C'(S) are the same.

    OK, this is confusing, so let us go through it slowly step by step and figure out what it means. To get to the punch line first, this condition says that the society's eventual identification of best elements in an environment is going to be determined solely by the rankings by the individuals of the alternatives in that environment, and not by the rankings by the individuals of alternatives not in the environment. [Remember, the Environment, S, is a subset of all the possible alternatives.] Now, take the condition one phrase at a time. First of all, suppose we have two different sets of individual rankings of all the alternatives. The first set of rankings is the Ri [there are as many rankings in the set as there are individuals -- namely, the first individual's ranking, R1, the second individual's ranking, R2, and so forth.] The second set of rankings is the Ri', which may be different from the first set.    

    Now, separate out some subset of alternatives, which we will call the Environment S, and focusing only on the alternatives in S, take a look at the way in which the individuals rank those alternatives, ignoring how they rank any of the alternatives left out of S. If the two sets of individual orderings, Ri and Ri', are exactly the same for the alternatives in S, then when the Social Welfare Function cranks out a social ranking, R, based on the individual orderings Ri and a social ranking, R', based on the individual orderings Ri', Condition 3 stipulates that the set of best elements [The Social Choice set] will be the same for R and for R'.

    Whew, that still isn't very clear, is it? So let us ask the obvious question: What would this Condition rule out? Here is the answer, in the form of an elaborate example. Just follow along.

    Suppose that in the 1992 presidential election, there are just three voters, whom we shall call 1, 2, and 3. Also, suppose there are a total of four eligible candidates: George H. W. Bush, Bill Clinton, H. Ross Perot, and me. Now suppose there are two alternative sets of the rankings of these four candidates by individuals 1, 2, and 3.

Ri:     Individual 1: Wolff > Clinton > Bush > Perot

    Individual 2: Bush > Perot > Wolff > Clinton

    Individual 3: Wolff > Clinton > Bush > Perot

Ri':    Individual 1: Clinton > Bush > Perot > Wolff

    Individual 2: Bush > Perot > Clinton > Wolff

    Individual 3: Clinton > Bush > Perot > Wolff

    The crucial thing to notice about these two alternative sets of rankings is that they are identical with regard to the environment S = (Bush, Clinton, Perot). The only difference between the two sets is that in the second set, Wolff has been moved to the bottom of everyone's list. [The voters find out I am an anarchist.]

    Now let us consider the following Social Welfare Function: For each individual ranking, assign 10 points to the first choice, 7 points to the second choice, 3 points to the third choice, and 2 points to the fourth choice. Then, for any Environment, S, selected from the totality of available alternatives, determine the social ranking by adding up all of the points awarded to each alternative by the individual rankings. Got it?

    Go ahead and carry out that exercise. If you do, you will find that for the first set of rankings, the Ri, and for the Environment S = (Bush, Clinton, Perot). the SWF gives 16 points to Clinton, 16 points to Bush, and 11 points to Perot. So, C(S), the society's decision as to which candidates are at the top, is (Clinton, Bush), because they each have the same number of points, namely 16. But if you now carry out the same process with regard to the second set of individual rankings, the Ri', and the same Environment S, you will discover that the SWF assigns 23 points to Clinton, 24 points to Bush, and 13 points to Perot, which means that C'(S) is (Bush). So the social choice in the Environment S has changed, despite the fact that the relative rankings of the elements in S have not changed, because of a change in the rankings of an element not in S, namely Wolff. And this is just what Condition 3 rules out. It says that the Social Welfare Function cannot be one that could produce a result like this.

    All of us are familiar with this sort of problem from sports meets or the Olympics. When we are trying to decide which team or country has done best, we have to find some way to add up Gold medals and Silver medal and Bronze medals, and maybe fourth and fifth places as well. And, as we all know, you get different results, depending on how many points you award for each type of medal. Arrow's Condition 3 rules out SWFs like that.

Condition P: If xPiy for all i, then xPy. This just says that if everyone strongly prefers x to y, so does the society. This is a very weak constraint on the SWF.

Condition 5: The Social Welfare Function is not dictatorial.

    Remember the definition of "dictatorial" above. This rules out "l'état c'est moi" as a Social Welfare Function.

    So, we have the definitions, etc., and we have the four Conditions that Arrow imposes on a Social Welfare Function. Remember that a Social Welfare Function is defined as a mapping that produces a social ranking that satisfies Axioms I and II. Now Arrow is ready to state his theorem. It is quite simple:

    There is no Social Welfare Function that satisfies the four Conditions.

Friday, July 16, 2010

COLLECTIVE CHOICE THEORY FIRST INSTALLMENT

Part Four: Collective Choice Theory

    Collective Choice Theory is the theory of how one selects a rule to go from a set of individual preference orders over alternatives available to a society of those individuals to a collective or social preference order over those same alternatives. [Or, as they say in the trade, how to "map a set of individual preference orders onto a social preference order."] There is a long history of debates about how to make social or collective decisions, going back at least two and a half millennia in the West. The simplest answer is to identify one person in the society and stipulate that his or her preference order will be the social preference order. L'etat, c'est moi, as Louis XIV is reputed to have said. A variant of this solution is the ancient Athenian practice of rotating political positions. One can also choose a person by lot whose preferences will thereupon become the social preference. A quite different method is that used by the old Polish parliament, which consisted of all the aristocrats in the country [there were quite a few, the entry conditions for being considered an aristocrat being low]. Since each of them thought of himself as answerable only to God, they imposed a condition of unanimity on themselves. If as few as one Polish aristocrat objected to a statute, it did not become law.

    These rules for mapping individual preference orders onto a social preference order, unattractive as they may be on other grounds, all have one very attractive feature in common: They guarantee that if all of the individual preference orders are ordinal orderings, which is to say if each of them is complete, reflexive, and transitive [you see, I told you we would use that stuff], then the social preference order will also be an ordinal ordering, and that is something you really, really want. You want it to be complete, so that it will tell you in each case how to choose. And you want it to be transitive, so that you do not get into a situation where your Collective Choice Rule tells the society to choose a over b, b over c, and c over a.

    To sum it all up in a phrase, the aim of Collective Choice Theory is to find a way of mapping minimally
rational individual preferences onto a minimally rational social preference.

    For the past several hundred years, everybody's favorite candidate for a Collective Choice Rule has been majority rule. This is a rule that says that the social preference between any two alternatives is to be decided by a vote of all those empowered to decide, with the alternative gaining a majority of the votes being preferred over the alternative gaining a minority of the votes. Should two alternatives, in a pairwise comparison, gain exactly the same number of votes, then the society is to be indifferent between the two.

         Enter the Marquis de Condorcet, who published an essay in 1785 called [in English] Essay on the Application of Analysis to the Probability of Majority Decisions. In this essay, Condorcet presented an example of a situation in which a group of voters, each of whom has perfectly rational preferences over a set of alternatives, will, by the application of majority rule, arrive at an inconsistent group or social preference. This is, to put it as mildly as I can, a tad embarrassing. Indeed, it calls into question the legitimacy of majority rule, which lies at the heart of every variant of democratic theory that had been put forward at that time, or indeed has been put forward since.

    Let us take a moment to set out the example and examine it. In its simplest form, it involves three voters, whom we shall call X, Y, and Z, and three alternatives, which we shall call a, b, and c. We may suppose that a, b, and c are three different tax plans, say. Let us now assume that the three voters have the following preferences over the set of alternatives S = (a, b, c).

    X prefers a to b and b to c. Since X is minimally rational, he also prefers a to c.

    Y prefers b to c and c to a. Since she is also minimally rational, she prefers b to a.

    Z prefers c to a and a to b. As rational as X and Y, she naturally prefers c to b.

    Now they take a series of pairwise votes to determine the collective or social preference order among the three alternatives. When they vote for a or b, X and Z vote for a, Y votes for b. Alternative a wins. When they vote for b or c, X and Y vote for b, Z votes for c, alternative b wins. Now, if the social ordering is to be transitive, then the society must prefer a to c. What happens when X, Y, and Z choose between a and c? X prefers a to c. But Y and Z both prefer c to a. So the society must, by majority rule, prefer c to a. Whoops. The society's preference order violates transitivity.

    And that is the whole story. The selection of a social or collective preference order by majority rule cannot guarantee the transitivity of the social preference order, and therefore does not even meet the most minimal test of rationality. There are, of course, lots and lots of sets of individual preference orders that generate a consistent social preference order when Majority Rule is applied to them. The problem is that here is at least one, and actually many more, that are turned by Majority Rule into an inconsistent preference order.

    If you have never encountered this paradox before [the so-called paradox of majority rule], you may be inclined to think that it is a trick or a scam or an illusion. Alas, not so. It is just as it appears. Majority Rule really is capable of generating an inconsistent social preference ordering.

    All of this was well known in the eighteenth century, and was, as we shall see later on, the subject of some imaginative elaboration by none other than the Reverend Dodgson, better known as Lewis Carroll. Enter now the young, brilliant economist Kenneth Arrow in the middle of the twentieth century. Coming out of a tradition of economic theorizing called Social Welfare Economics, to which a number of major figures, such as Abram Bergson, had contributed, Arrow conceived the idea of analyzing the underlying structure of the old Paradox of Majority Rule and generalizing it. The result, which he presented in his doctoral dissertation no less, was The General Possibility Theorem. Arrow published the theorem in 1951 in a monograph entitled Social Choice and Individual Values.

    Another great economist and fellow Nobel Prize winner, Amartya Sen, in 1970 published Collective Choice and Social Welfare, in which he generalized and extended Arrow's work in astonishing ways. Sen's book is difficult, but it is simply beautiful, and deeply satisfying. I strongly urge you, if you have a taste for this sort of thing, to tackle it. Sen has written widely and brilliantly on a host of extremely important social problems, including economic inequality, famine, and the demographic imbalance between men and women in the People's Republic of China. His little series of Radcliffe Lectures, published in 1973 as On Economic Inequality, is the finest use of formal methods to illuminate and analyze a social problem of which I am aware. It is a perfect example of the proper use of formal methods in social philosophy, and as such deserves your attention.

    In Collective Choice and Social Welfare, Sen gives a simpler and more elegant proof of Arrow's General Possibility Theorem. Nevertheless, I have chosen in this blog to expound Arrow's original proof. Let me explain why. It often happens that the first appearance of an important new theorem is somewhat clumsy, valid no doubt, but longer and more complicated than necessary. Later theorists refine it and simplify it until what took many pages can be demonstrated quickly in a few lines. Sometimes, this development is unambiguously better, but at other times, the original proof, clumsy though it may be, reveals the central idea more perspicuously than the later simplifications do. I find this to be true in the case of Arrow's theorem. Sen's simplification serves several purposes, not the least of which is to set things up formally for his extremely important extension and elaboration of Arrow's work. Therefore, I urge you to look at it, once you have worked with me through Arrow's original proof.

    Now let us begin. This is going to take a while, so settle down. Before we get into the weeds, let me try to explain in general terms what Arrow is doing. He asks, in effect, what are the underlying general assumptions of majoritarian decision making? What is it about voting with majority rule that appeals to us? He identifies five conditions or presuppositions [later reduced and simplified to four] that capture the logic of majority rule in a general way, and then shows that no way of making collective decisions that satisfies all four of them guarantees that the resulting social or collective choice will be consistent. This way of thinking about the problem accomplishes three things simultaneously. First, it unpacks majority rule voting into its component parts so that we can look at it and understand it better. Second, it generalizes the Paradox of Majority Rule so that we realize we cannot avoid it simply by tweaking Majority Rule a bit [for example by requiring a two-thirds majority.] And finally, it allows us to see just exactly what Majority Rule does not do -- in other words, it gives us insight into what would be totally different ways of making collective decisions.

    We start with a series of assumptions, definitions, and notational conventions, some of which are already familiar to you from the opening segments of this general tutorial. This is going to be tedious, but learning these up now will make it infinitely easier to follow the proof. Here they are:

(a) We start with a set of mutually exclusive alternatives, x, y, z, ..... These may be all of the possible candidates in an election [i,.e., every single person who is eligible to hold office under the rules governing the election], every possible tax scheme that might come before Congress, all of the various possible decisions a City Council might take concerning zoning regulations, and so forth. The point of the phrase "mutually exclusive" is to rule out, for example, "Obama" and "Obama or Clinton" as two of the available alternatives.

(b) On any give occasion when a decision is to be made, there is a subset, S, of the available alternatives, which will be called The Environment. This might be, for example, the relatively small number of people who have stated publicly that they would like to be elected to that office, or all the people who have formed campaign committees, or all the people who survive the primary season and are on the final ballot. Each of these is a subset of all the people eligible to hold the office [not necessarily a proper subset -- i.e., not necessarily smaller than the total set of alternatives. All that is required is that S be included in the set of all alternatives, not that it be smaller than that set].

(c) There is a set of individuals ["voters"], identified by numerical subscripts, 1, 2, 3, 4, ....

(d) Each individual is assumed to have a complete, transitive ranking of the entire set of alternatives, which we indicate using the notation introduced earlier -- the binary relations R, I, and P. Just to review, xRiy means that individual i considers alternative x to be as good as or better than alternative y. xPiy and xIiy are derived from R in the way indicated in the opening segments of this tutorial. What we are aiming for, of course, is a collective or social ranking, and that is indicated by the same letters, R, P, and I without the subscripts. So xPy means that the society prefers s to y. The whole point of this exercise is to start with complete, transitive individual rankings of the alternatives and then see whether there is any way of going from the individual rankings to a social ranking that satisfies certain conditions [see below] and results in a social ranking that is complete and transitive.

(e) Ri all by itself refers to individual i's ranking of the entire set of alternatives, x, y, z, .... Correspondingly, R all by itself refers to the society's ranking of the entire set of alternatives.

(f) We shall have occasion to refer to different possible rankings, by an individual i, of the set of alternatives. We will indicate these different rankings by superscripts. So, for example, Ri is one ranking by individual i of the entire set of alternatives. Ri' is a second ranking. Ri'' is a third ranking. And Ri* is a fourth ranking. A ranking Ri can be thought of either as a list showing the way individual i ranks the alternatives, including ties [indifference], or as a set of all the ordered pairs (x,y) such that xRiy.

(g) A Social Welfare Function [ an SWF ] is a function that maps sets of individual rankings onto a social ranking. Such a mapping function qualifies as an SWF just in case both the individual rankings, the Ri, and the social or collective ranking, R, satisfy Axioms I and II below -- which is to say, just in case the rankings, both individual and social, are complete and transitive.

(h) A Social Welfare Function is said to be Dictatorial if there is some individual i such that, for all x and y, xPiy implies xPy regardless of the orderings of all of the individuals other than i. Thus, in particular, to say that an SWF is dictatorial is to say that there is some individual who can impose his or her will on the society with regard to the choice between any pair, x and y, even if everyone else in the society has the opposite preference as between those two alternatives.

     (i) Finally, we define something called a Social Choice Function [ symbolized as C(S).] C(S) is the set of all alternatives x in the Environment S such that for every y in S, xRy. In other words, C(S) is the set of top alternatives or best alternatives in S. Quite often, C(S) will contain only one alternative, the one that the society prefers over all the others. But it may include more than one if the society is indifferent as among several best alternatives.

    Those are the nine definitions and stipulations. The key new ones that we have not met before are S, the set of available alternatives, R, the social ranking, SWF, a Social Welfare Function, and C(S), the Choice Function. Now Arrow lays down two Axioms governing the social ordering, R. These are:

Axiom I: For all x and y, xRy or yRx [Completeness]

Axiom II: For all x, y, and z, if xRy and yRz then xRz. [Transitivity]

    O.K. So much for the preliminary throat clearing. I want you to go over these definitions and stipulations until you are comfortable with them. The proof is going to be a formal argument couched in terms of these symbols and appealing to these assumptions and axioms. You will find it impossible to follow if you do not have a solid grasp on these preliminary definitions and so forth. While you are doing that, I want to talk for a bit about several important points that are implicit in what we have just laid down, but may not be obvious.

Wednesday, July 14, 2010

Rawls Last Installment

The first thing an individual in the Original Position must do when confronted with a choice of basic organizational rules for society is to decide how well or badly off she is, or was, before entering the Hall of Justice. [I shall simply stipulate that our representative person is female, but of the course the person does not know this, or indeed anything else of a particular nature about him/her self.] Since Rawls says that she is rationally self-interested, and is prepared to enter into the bargaining game because she believes that a satisfactory outcome will be to her advantage, she clearly needs to know what her baseline situation is. Otherwise, she cannot make a judgment as to whether a proposed rule will make her better off. Remember: she not only does not know who in particular she is or where in her society she is situated. She also does not know what stage of history she is located in.

    Faced with the necessity of stipulating a pre-bargain baseline [defined, we may suppose, simply by some specified amount of Primary Goods -- this whole thing just gets hopelessly complicated if we try to flesh out her situation in any more realistic manner], she really has only three options. For each possible stage of history in which she might be located, she can either adopt the premise that she is the worst off representative person in that society; or she can adopt the premise that she is the best off representative individual; or she can carry out an expected utility calculation, assigning some level of Primary Goods and some probability to every representative position in the society, and then multiplying the two and summing the results, In this third case, she will say to herself something like this: "There are seven representative positions in the society; fifteen percent of the people are in the first, ten percent in the second, etc etc. The first position has so and so much of the Primary Goods assigned to it, the second such and such amount, and so forth; with no more information than that I am one of the people in the society, I conclude that I have a fifteen percent chance of being in the first position, a ten percent chance in the second position, and so forth. Assuming that I know what my cardinal utility function is for Primary Goods, I can now carry out my expected utility calculation."

    Sigh. I told you this was going to be messy. I am pretty sure, from correspondence I had with Jack, that he is aware of a good deal of this, but I do not think he ever fully appreciated how deeply it undercut his central claim that he was advancing a theorem. At this point, Rawls says that a rational person, recognizing how important the choice is that she is about to make, will adopt an extremely conservative way of evaluating alternatives. What does this mean?

    Well, the first thing it means is assuming that outside the Hall of Justice, in the real world, she is one of the persons occupying the least advantaged representative position in society. Why is this conservative? Because if she assumes that she is in fact well off in the real world, she will be correspondingly less willing to make a deal, and this threatens to leave her utterly disadvantaged should the optimistic assumption about herself prove false. She must protect herself against the chance that she is one of society's poor, and the best way to do this is to agree to inequalities of any sort only if they work to the advantage of those least well off.

    But reasoning in this fashion, she might be tempted to carry out some sort of expected utility calculation and opt for a set of principles that maximizes the average utility that each representative person will enjoy. To be sure, that can be risky, since a higher average overall might be compatible with a lower utility to the least well off. In an expected utility calculation, that risk might be compensated for by a chance at a very much higher payoff to the better off representative positions.

    Rawls now argues that the rational individual under the Veil of Ignorance will reject expected utility calculations and instead opt for the extremely conservative, and also extremely controversial, "maximin" rule proposed by von Neuman. On page 163 of my book [see the chapter to which I have linked], I quote Rawls' reasons for adopting this rule. Here is what he says: "There are three chief features of situations that give plausibility to this unusual rule... The situation is one in which a knowledge of likelihoods is impossible or at best extremely insecure...The person choosing has a conception of the good such that he cares very little, if anything, for what he might gain above the minimum stipend that he can, in fact, be sure of by following the maximin rule. It is not worthwhile for him to take a chance for the sake of a further advantage, especially when it may turn out that he loses much that is important to him.... The rejected alternatives have outcomes that one can hardly accept. The situation involves grave risks." [All four passages from Rawls, p. 154]

    In my book, I have given a formal analysis of these claims, complete with nifty diagrams, but I want here to step back and try to get a sense of what Rawls is really talking about. Remember, first of all, that Rawls is not talking about the quantity of Primary Goods that the various principles of justice offer as possibilities, but rather about the utility that the utility function of the individual under the Veil of Ignorance associates with these various amounts of Primary Goods. The distinction is essential for understanding what Rawls is saying.

    Concretely, Rawls is claiming that the rational individual under the Veil of Ignorance will say to herself: "If I opt for a system of social organization that holds out the possibility of vast wealth for a few, but that fails to protect those at the bottom from absolute penury, I am risking ending up in a disastrous situation, one that "involves grave risks." But all I stand to gain is the chance at one of the top spots, even though I "care very little, if anything, for what [I] might gain above the minimum stipend that [I] can, in fact, be sure of by following the maximin rule."

    Fair warning: I am now going to say something that is mean-spirited and snarky, but I really do not know how else to get at what is going on in this argument. I apologize if I offend anyone. Here goes:

    What sort of person says to himself or herself what the individual in the Original Position, according to Rawls, says? Not just a rational person. There is nothing formally irrational about being willing to risk utter penury for a chance at fabulous wealth. That is just a matter of having a utility function of a particular shape[one that is, over a certain range, monotonically increasing rather than decreasing.] Would Gordon Gekko think this way? [If there is anyone who does not recognize the name, Gordon Gekko is the main character of the 1987 film, Wall Street, starring Michael Douglas. If you haven't seen it, by all means get it from NetFlix.] Of course not. But Gordon Gekko is not formally irrational. He just places a very high value on vast wealth and has a very high tolerance for risk. What about Picasso? I think not. If you offered Picasso a chance at artistic immortality, with penury and misery as the alternative if he turned out not to have real talent, I think he would have grabbed the chance with both hands. In fact, of course, he did.

    No, the sort of person who would reason as Rawls thinks the individual in the Original Position would is a tenured professor -- someone who has a comfortable albeit modest lifestyle that is absolutely assured against any risks, someone who has perhaps turned down other careers offering much larger rewards but also "involving grave risks." In short, the sort of person who would reason as Rawls thinks the individual in the Original Position would is ... John Rawls.

    Strip away all the talk about theorems, all the lovely filigree of philosophical elaboration, all the Reflective Equilibrium and Strains of Commitment and allusions to Game Theory, and you have a simple apologia pro vita sua.

    If the Representative Individual in the Original Position is an academic at a good American university or college that offers life tenure and a comfortable middle class life, then I think it is quite likely that he or she would opt for Rawls' two principles. They guarantee a continuation of that pleasant life style, combined with a virtuous but really cost free concern for the poor downtrodden denizens of the Inner City [the least well off representative individuals].

    Now, that is just about as mean-spirited as I have ever been in print [though not, I am afraid, in person], but what else can one conclude if one takes Rawls' theory seriously and tries to think through what it really means?

    The time has come to step back from the details of Rawls' discussion and try to get some perspective on what is, when all is said and done, the most important contribution to political philosophy of the past hundred years and more. I observed at the beginning of these remarks that Rawls offered his very new theory at a time when Anglo-American Ethical Theory was mired in an antinomy -- a several decades long face off between Intuitionism and Utilitarianism. Rawls invited us to get past that stalled historical moment by making use of ideas drawn from Game Theory [and also from neo-classical economics, but that is another matter.] If he had simply offered his Two Principles as an alternative to, or perhaps more accurately as a fusion of the best parts of, Intuitionism and Utilitarianism, there is no question that his proposal would have commanded considerable attention. The elegance of his discussion of Utilitarianism and the interesting and suggestive detail of the fully elaborated version of his proposal would, I am sure, have generated a lively discussion among philosophers, political theorists, and others.

    But what made Rawls' theory stand out as deserving of what constitutional lawyers call heightened scrutiny was his claim to be able to establish his two principles as the solution of a bargaining game. Now, even if this thesis could be sustained, it would still be open to readers to reject Rawls' claim that the solution of such a game ought to be considered the principles of social justice. But a genuine proof of Rawls' theorem would have vaulted his theory to an entirely unique status in ethical and political theory. Such a theorem would have taken its place beside Kenneth Arrow's General Possibility Theorem as a major result of formal analysis. [I remain convinced, in the absence of any textual or anecdotal evidence whatsoever, that this is exactly what Rawls dreamed of accomplishing.] This is why, both in my book and in these blog posts, I have focused almost exclusively on the logical status of the theorem that Rawls adumbrates in "Justice as Fairness," and continues to allude to as a theorem, albeit in a hedged manner, in Ðistributive Justice" and A Theory of Justice.

    I think I have demonstrated that the theorem is not valid, either in its original or in its revised form, or, more precisely, that it can only be made plausible by so many ad hoc adjustments, presuppositions, and qualifications that it loses its grip on our attention. I also think it is clear that the theory, as Rawls sets it forth in his book, covertly valorizes, without adequate argument, one particular substantive vision of the good society -- a vision some components of which I share, but for which Rawls fails to offer an argument.

    Well, this is twenty-four pages about Rawls, which is enough, I think, for this blog. I will turn my attention next to the single most important formal result in the application of formal methods to political philosophy: The General Possibility Theorem of Kenneth Arrow. My tone will change dramatically, as you will discover. No sniping or snarking, no ad hominem arguments. Arrow's result, like von Neuman's Fundamental Theorem, is a genuine triumph, and I shall do my best in expounding it to make its logical structure clear.

Monday, July 12, 2010

RAWLS FOURTH INSTALLMENT

Why the change? Pretty clearly, it is required by the special conditions imposed by the Veil of Ignorance. Since the parties in the Original Position now do not know who they are, they are pretty well forced either to carry out expected utility calculations over the entire range of positions in the society, some one of which each of them will actually turn out to occupy when they leave the Hall of Justice and regain their knowledge of who they are, or else to adopt the conservative assumption that they occupy the lowest position, and bargain to improve its allocation of Primary Goods. As we shall see, this is Rawls' version of the conservative rule proposed by von Neuman, to maximize one's security level.

    All this goes by so quickly in Rawls' exposition of the mature form of his theory that unless you are paying real close attention, you may not notice how wildly implausible, or even downright impossible, it all is. This is the form of the theory that everyone is familiar with, but people usually do not have any coherent idea why Rawls has made all of these very powerful stipulations. The reason, as I have indicated, is that each element of the final theory is designed to meet an objection to an earlier form of the theory.

    So where does all of this revision leave us? Well, first of all, something odd has happened along the way, as Rawls has altered his description of the choice situation to meet and overcome the difficulties with the first formulation. The original idea was that the parties to be governed by the agreed upon foundational rules would confront one another and bargain. The parties were assumed to be rationally self-interested, but with differing interests and desires. Rawls' central idea was that if to this premise of rational self-interest we added only one additional premise -- the willingness of the parties to abide by a set of rules arising from the bargain, the willingness to take that one step beyond self-interest to something resembling what is involved in having a morality -- then we could prove that the one and only set of rules on which they would self-interestedly settle would be his "two principles."

    But if we think about it for a moment, we will realize that after the revisions, Rawls no longer has a Bargaining Game that looks anything like this. Since the players have been stripped of any individuating features that might distinguish them from one another [such as differences in tastes or talents, or indeed even differences in which stage of human history they happen to be located in], there are no rational grounds on which any two of them could reason differently from one another about the choice of the basic structure and rules of the society in which they will find themselves when they emerge from the Veil of Ignorance. In short, what began as a problem in Bargaining Theory has morphed into a problem in the Theory of Rational Choice. [This is one of the reasons why Rawls tended to move toward what he himself called the Kantian Interpretation" of his theory. But that really does get us too far afield.]

    Before addressing the central question, viz., are these two principles thus revised, the solution to the Bargaining Game, thus altered, there is one subsidiary matter I should like to take up. Rawls says that although the parties in the Original Position under the Veil of Ignorance have temporarily forgotten who they are, what their specific desires are, and where they are located in history and in the structure of their society, they do retain a knowledge of the "general facts about human society." Each of these individuals, Rawls elaborates, understands "political affairs and the principles of economic theory," as well as "the basis of social organization and the laws of human psychology." In my book, Understanding Rawls, I argued that this is an epistemologically impossible state of affairs. There is not time or room here to repeat what I have said there [another shameless plug for my book :) ], but I think it is worth indicating the line of argument that I develop there.

    The first thing to be clear about is that under the Veil of Ignorance, the individuals in the Original Position do not even know in which stage of human history they are located. This fact leads Rawls into an extremely interesting discussion about the appropriate rate of savings that should be chosen as part of the basic socio-economic structure being negotiated. Debates about the social rate of savings are familiar to economists, but have been virtually absent from the political philosophy literature. It is greatly to Rawls' credit that he recognized this and introduced the subject into his theory. For those who are unacquainted with the discussion, the central issue is this: The capital required for future economic activity [seed corn, machinery, Research and Development, and their monetary equivalents] must be obtained from current production by somehow imposing limits on consumption -- eat all the corn this season, and there is no seed for next season's crop. Simple prudence dictates that people this year save for next year. But what shall we say about the responsibility of people in this generation to save for generations as yet unborn? A high, self-denying rate of social savings, such as that now being enforced by the Chinese government, will make possible an explosion of production in future generations, to the manifest benefit of those who are then alive. But that future production will come at the expense of this generation, which will have to deny itself some measure of present consumption.

    From the perspective of Rawls' theory, the question becomes: Under the Veil of Ignorance, what rate of savings will rationally self-interested individuals choose to impose upon themselves once they emerge from the Veil and discover which generation of their society's evolution they are actually located in? I encourage readers interested in this subject to take a look at Rawls' discussion.

    But getting back to the epistemological issue, the individuals in the Original Position are presumed to know the general facts of nature, society, economy, and human psychology, and even to know the broad outlines of the historical evolution of societies, but not to know where in that evolutionary process they are themselves located. Rawls clearly thinks it is possible for someone to be in this particular epistemological position. I think it is not. Why not?

    First of all, the individuals in the Original Position are blocked from accessing certain individuating facts about themselves, but they have not lost their powers of reason. To put the point simplistically, if the Veil has enabled them to retain the knowledge that All men are mammals and the knowledge that All mammals are animals, then their unimpaired powers of reason will allow them to conclude that All men are animals. Somewhat more to the point, if they know the standard theorems concerning the relation of supply to demand in the determination of price in a capitalist economy based on the production of commodities for sale in the marketplace, then they will be able to infer that their society has undergone the transition from Feudal to Capitalist social relations of production, because until such a transition has taken place, individuals do not even possess the concepts that are employed in the formulation of those economic laws. What is more, if, as I believe, capitalist social relations of production systematically mystify the underlying structure of exploitation on which capitalist profit rests, so that people mistakenly but inevitably perceive those relations as the expression of eternal and immutable economic laws, then only someone enmeshed in a capitalist society and economy will make the mistake of thinking that there are "laws of supply and demand."

    Now, maybe I am right about that, and maybe I am wrong. But by building these assumptions into the structure of the bargaining game from which he hopes to extract the principles of justice, Rawls has begged all of the questions that might be raised by someone like me ["begged" in the proper use of that term -- i.e., assumed what is to be proved]. This is one more example of my general claim that the misuse of formal methods allows authors to present their ideologically laden assumptions as value-neutral elements of a formal analysis.

    Let us now return to the central question: Would the individuals situated under the Veil of Ignorance in the Original Position coordinate on Rawls' Two Principles of Justice as revised in A Theory of Justice? This question is much more difficult to answer now than it was with regard to the first form of the theory. Even to make the question determinate enough to grapple with it we must make a considerable number of assumptions and specifications with regard to matters that Rawls either does not discuss or else leaves up in the air.

    At this point, in order to make this manageable, I must ask you to consult the chapter from my book, a link to which was posted earlier on this blog. I will discuss the problems in general terms, and leave it to each of you to read my detailed analysis in that chapter.

Friday, July 9, 2010

RAWLS THIRD INSTALLMENT

The major changes to the bargaining game introduced in "Distributive Justice" and carried over into A Theory of Justice are rather dramatic. They are four in number:

    (1) Rawls introduces the famous Veil of Ignorance, which is a brilliant literary device designed to capture what we mean when we say that judges must be "disinterested" -- i.e., that they must make decisions without taking into consideration their own interests or situation. This takes care of the problem that some participants in the Bargaining Game are, and know they are, among the more talented members of society, while others are, and know they are, among the less talented. But this fix comes with its own problems.

    Stripped of any knowledge of who they are, the players in the Bargaining Game now have no reason at all to bargain, or indeed to do anything else. This will be clear if we keep in mind the analogy to judges. Judges, in their judicial capacity, are not supposed to have an agenda [think Confirmation Hearings for Supreme Court nominees and all the blather about "activist justices" who "make law from the bench."] So if the individuals under the veil of ignorance know nothing at all about who they are, they can have no ends, no purposes, save perhaps the goal to render decisions fairly when they are presented with cases. Hence these individuals have no reason to bargain about anything at all. The self-interest Rawls has equipped them with is vacuous. Rawls is thus forced to endow the individuals in the Original Position with some sorts of purposes, specific enough to make them care what they get from the agreement being hammered out, but not so specific as to recreate the problems that invalidated the first version of his theory.

    (2) Rawls' solution is to state that the individuals in the Original Position know that they have Life Plans. And since it wouldn't do to allow them to have the life plan of a religious hermit, for example [because then they would not care how much stuff they got out of the bargain, or even whether they had civil rights and protections], Rawls stipulates that the Life Plans posited for the individuals in the Original Position require for their accomplishment certain rights and abilities, powers and endowments [i.e., some stuff]. Bu this creates a new raft of problems, of an especially intractable sort, because the sorts of rights and abilities, powers and endowments required for the fulfillment of one Life Plan may be quite different from those required for the fulfillment of a different Life Plan. If your Life Plan is to become a Professor of Philosophy, then access to higher education for all those sufficiently talented is going to loom large in your budget of things you are bargaining for. But if your Life Plan is to become a champion surfer, higher education may drop way down in your list of desiderata.

    [Notice, by the way, that built into Rawls' conception of Life Plans is a particular substantive historical, and cultural, and social conception of individual personality and the good life. I do not have time to go into this in the detail that it deserves, so I will simply refer you to the classic discussion in Karl Mannheim's Ideology and Utopia, Chapter IV, of the chiliastic, liberal-humanitarian, conservative, and socialist-communist forms of the Utopian Mentality and the orientation to time itself. Rawls, to put it in shorthand, assumes that everyone in the Original Position has a liberal-humanitarian orientation toward time, which is simply part of the larger fact that his entire theory is an ideological rationalization of capitalist social democracy. But I digress.]

    To get over the problem posed by the diversity of possible Life Plans, so that he can get back to his theorem, Rawls now makes another assumption, and this one is, technically speaking, a whopper.

    (3) since they do not know which particular Life Plans they have, Rawls asserts [he never offers anything remotely like an argument for it] that one can create an index of the heterogeneous basket of basic stuff that anyone would need to pursue whatever Life Plan he or she might turn out to have, something Rawls calls an Index of Primary Goods He then simply assumes that everyone in the Original Position has positive, albeit declining, marginal utility for the Index of Primary Goods. He assumes this, not because it is plausible [he never offers any arguments for any of this], but because unless he assumes it he won't have a hope of invoking the tools and techniques of modern economic theory with which he thinks he can prove his "theorem."

    The problem of constructing an index of a heterogeneous assortment of rights and abilities, powers, and endowments is completely insoluble, as any honest economist will tell you. Indices like the Consumer Price Index or the Dow Jones Index are hopelessly flawed, and there is no way to fix them. Some of you may be quite familiar with this problem, others may have never heard of it before. I guess maybe I ought to say a word or two about the subject, even though it will slow us down some. The Consumer Price Index is constructed by putting together a list ["a market basket"] of consumer goods in specific proportions that is supposed to reflect the way most Americans spend their household income in a specified time, say a month: So much housing, so many pounds of potatoes, so many lamb chops, so many visits to the doctor, etc. Samples are then taken, and averaged out, of what these items are selling for in various stores, doctors' offices, etc, around the country. The same market basket of goods and services is then priced a month later. If the total cost of the market basket has increased by 1%, then it is said that there has been a 1% increase in the CPI, or that there has been a month to month inflation rate of 1%.

    There are a gazillion problems with this index, all of which are totally unfixable. For example, suppose you have a thirty year fixed rate mortgage on your house. In that case, if housing costs rise dramatically [as they did in the 70's and 80's, for example], the Consumer Price Index, reflecting that rise, may shoot up dramatically. But you won't actually experience that rise, because a major component of your market basket of goods, namely housing, is fixed. The same sort of problem arises with regard to every element in the "market basket." The dramatic rise in health care costs will not affect you so long as you are healthy, but will affect you if you have a special needs child. if you are a vegetarian, a steady decline in the price of meat won't have any effect on your pocketbook. When I was a young man living in Cambridge, Massachusetts, lobster was cheap at the supermarket and steak was expensive. Now, when I shop for dinner, steak is cheaper per pound than all but the least expensive fish. Once again, Rawls unconsciously [I think] builds intro his supposedly universal theory the assumptions, presuppositions, and tastes of a particular social and economic class, which happens to be his.

    (4) The last major change is a revision in the statement of the so-called "Difference Principle." Instead of inequalities working to everyone's advantage, they must now work to the benefit of the least advantaged members of society. This is a very significant change However, Rawls introduces it not by saying that he has changed his mind, or has been compelled by the logic of the bargaining game to alter the theorem, but rather by saying that this is a more reasonable "interpretation" of the original form of the principle. I confess that I find this rather weird and creepy. Rawls treats his own two principles, which he made up, as though they had been inscribed on tablets by The Lord God Himself, and thus required us to interpret them rather than change them. This is one of the strangest features of Rawls' entire discussion.

Wednesday, July 7, 2010

APPLICATIONS RAWLS SECOND INSTALLMENT

The second principle says that " inequalities are arbitrary unless it is reasonable to expect that they will work out for everyone's advantage." Remember that in the original version of Rawls' principles, it is inequalities in a practice that are being referenced -- differential salaries, for example, or differences in the perks associated with one of the roles in the practice. The core idea is that economic inequalities may actually result in an increase in total social output, for example by attracting especially talented people to positions demanding highly skilled workers. Inequalities may also motivate young people to acquire time consuming and demanding skills whose deployment in more highly compensated positions will once again increase total output. Higher wages are required to attract the talented workers or to persuade them to spend the time and money acquiring the skills. If there is something left over from the increased output after the skilled workers have received sufficient additional compensation to motivate them, then the remainder -- what we might call an "inequality surplus" --can be spread around to the rest of the society, making everyone better off than he or she would have been in a society that enforces equal compensation at the price of a universally lowered standard of living.

    This is really the core idea in Rawls' entire theory of social justice. It is, we may note, the standard Sociological rationale for the extreme inequality of modern capitalist society. I like to think of it as the Brain Surgery argument -- to wit, "If you have to have brain surgery, do you want to be operated on by someone who is paid no more than a burger flipper at Wendy's?" The idea roughly is this: If everyone were paid the same wage -- say something above what is now Minimum Wage -- no one would have any particular motivation to swap the job of burger flipper or ditch digger or garbage collector for the job of corporate executive or brain surgeon or professor, assuming that those jobs had been stripped of their various perks as well as of their higher salaries [corner offices, people who call you "sir" or "ma'am," and so forth.] But society needs talented corporate executives and well-trained brain surgeons and professors. Otherwise the Gross Domestic Product will be sub-optimal and everyone will suffer. So efficiency demands that we slap big salaries on those jobs to motivate some of the more promising burger flippers and garbage collectors to trade in their jobs for brain surgery. If we manage things skillfully, we will pay them just enough to drag them away from their spatulas and garbage trucks and into the executive suites and hospitals. Those salary increases will come out of the GDP, of course, but there will be enough left over to raise the pay of all the remaining burger flippers and garbage collectors. So, assuming no one is envious, and resents the higher salaries of the brain surgeons even though the productivity of the brain surgeons is raising the wages of the burger flippers, everyone will be in favor of this inequality. [This is the reason for the non-envy clause in Rawls' theory, in case you ever wondered.]

    I have spelled this out at length because thus explicated, it is so wildly implausible. As a description of what motivates people in a modern capitalist society to pursue one career path rather than another it is so tone-deaf sociologically and psychologically as to sound like a Jon Stewart send-up. But that is not the focus of these remarks, so let us leave that to one side.

    One of the purposes of the Difference Principle, so called, is supposed to be to allow us to adjudicate complaints against a scheme of unequal compensation by showing that the inequality works to everyone's advantage. But "to everyone's advantage" is, grammatically speaking, a comparative rather than a superlative. To everyone's advantage compared to what? This is a good deal harder to answer than it might seem at first glance.

    Presumably, the answer is that the scheme works to everyone's advantage compared to the same social system with equal compensation, if indeed one can even imagine the same social system without the inequalities. [Would a corporation be the same institution if everyone made the same wage? Would a hospital be?] But there are almost certainly a number of alternative schemes of unequal wages, each of which generates an Inequality Surplus adequate to make everyone better off, and each of which makes some positions better off and some positions worse off than those positions would be under a different inequality scheme generating a surplus. If that is so, considerations of Pareto Comparability do not permit us to say which scheme is preferable to which, even though each of them is preferable to the society without inequality.

    Assuming [what is in fact false] that all of these problems with the Two Principles can be solved, we are left with the central question: Would a group of rationally self-interested individuals faced with the circumstances of justice necessarily coordinate on the Two Principles? [By the way, for those who are not up to speed on all of this, the "circumstances of justice" are these: First, that the members of the society have something to gain from cooperation, if they can only agree; and Second, that their pre-agreement assets and powers are sufficiently equal to motivate them to seek common agreement. This is all standard Social Contract stuff, straight out of Hume, Hobbes, etc.] In short, we are faced with a proposed theorem in Bargaining Theory.

    As Rawls conceives the Bargaining Game, this is a multi-party game with full communication among players who are assumed to have cardinal utility functions invariant under affine transformations. Rawls never tells us how the game is played, nor does he even seem to think that he needs to do so. That is one of the odd things about his invocation of "theorems" in Game Theory. We are left to try to imagine for ourselves how the game would actually be played. I think we are meant to imagine something like this: The players sit in a circle in such a way that what each says is heard, and is known to be heard, by all. One player starts, and proposes a rule. [Say, the old Bill Cosby rule from early Sesame Street -- "All for one and everything for myself."] The next player either accepts the first player's rule, which of course she won't, or proposes a new rule. They go around the circle again and again, proposing foundational rules, until they succeed in making one complete circle during which everyone agrees to the same rule. That is then the solution to the game. Rawls says that after a bit "they will settle on [his] two principles." Is he right?

    Alas, no. There are two problems, one procedural, the other substantive. The procedural problem is that the bargaining game has no termination rule. There is no reason for the first player ["Everything for myself"] ever to stop proposing that rule. There is presumably some very, very small, but not zero, probability that sooner or later [probably later] the rest of the players will get tired or drop the ball and agree. Since there are no costs in the game associated with continuing to play it, none of the players has any incentive to "be reasonable."

    You can fix this glitch, of course, by imposing a time limit on the game. But that gives an asymmetrical advantage to the last player whose turn leaves just enough time to go around the circle once. When that moment is reached, the lucky player can propose a rule that makes everyone better off than having no rule at all, but advantages that player [such as "People with naturally curly hair get first dibs on all the nifty jobs," said by someone who has naturally curly hair]. Now, this is silly, right? But when you claim to be proving a theorem that is necessary, that is the sort of thing you have to take into account. This is an example of what I mean by wrapping yourself in the impressive language of Game Theory to make what you are doing sound impressive, while not actually engaging in Game Theoretic arguments. As Rawls says in "Justice as Fairness," "there remain certain details to be filled in, and various alternatives to be ruled out." Indeed.

    The second problem is substantive. The two principles proposed by Rawls would not win unanimous agreement from the players. The problem is this: While the players are faced by the circumstances of justice, and hence are roughly comparable in their powers and endowments, there are nevertheless significant differences among them in natural talents and abilities. Some of them will fare much better than others in a society in which "the positions ... are open to all." What is more, despite these differences in native intelligence or ability, all of them know these facts. Now, if you are one of the most talented members of society, you are going to be in favor of a structure of inequality in which you know quite well that you will be one of those ending up in the favored positions. But if you are not one of the talented, you will conduct an expected utility calculation and come to the conclusion that you might be better off with a system in which the better paid jobs are distributed by lot to anyone who meets certain minimum requirements [for example, a college degree, even with a low GPA]. The imposition of minimum requirements will suffice to generate some Inequality Surplus, but the allocation of the favored jobs by lot will work to the advantage of the less talented members of the society, who will thus have a shot at the higher paid jobs, something they would never have if those jobs were allocated strictly on the basis of a fair competition. [As Senator Roman Hruska of Nebraska said in 1970, defending the Supreme Court nomination of G. Harrold Carswell from the charge of mediocrity, "There are a lot of mediocre judges and people and lawyers. They are entitled to a little representation, aren't they?"]

    So Rawls' "theorem" is no theorem at all. I pointed this out in an article I published in The
Journal of Philosophy, and Jack's face fell when I told him about it at an annual APA meeting. But I went on to say that his subsequent essay, "Distributive Justice," changed his theory in a way that met all of my objections, and his face brightened. "Oh, that's all right then," he said before wandering off.

Monday, July 5, 2010

Rawls' A THEORY OF JUSTICE First Installment

Part IV Applications -- Rawls

    We come now to what might plausibly be considered the real payoff for all the technical thrashing about we have been engaged in: an extended analysis of the core argument in John Rawls' famous book, A Theory of Justice. Rawls' hauptwerk is widely considered the most important contribution to English language political theory of the past century, and is arguably the most influential work of philosophy written in the English language during that time. It is worth our while, therefore, to take the time to look at his central argument carefully and in detail. Thirty-three years ago I wrote a book-length examination of A Theory of Justice, called Understanding Rawls, published by Princeton University Press. Much of what I say here overlaps with what I said in that book, but my focus here is more narrowly on Rawls' attempt to apply Bargaining Theory to his subject. Those interested in a somewhat broader discussion are invited to hunt up my book and take a look. I am going to assume that everyone reading these words has some familiarity with Rawls' theory.

    The core of Rawls' work is a simple and rather lovely idea. In the middle of the twentieth century, Anglo-American ethical theory was stuck in what Kant, two centuries earlier, had called an antinomy. Utilitarians and intuitionists were locked in a death struggle, with each side more than capable of exposing the weaknesses of the other, but each unable to defend itself against the other's crushing arguments. Rawls had the idea that the conflict might be resolved by combining an old tradition of political philosophy -- social contract theory -- with a brand new field of mathematics and economics, Game Theory. Early in the development of the theory that eventually found its full-scale exposition in A Theory of Justice, Rawls claimed that he could prove a theorem in Bargaining Theory, and that the proof of that theorem would constitute a justification for the pair of principles which, he said, were or ought to be the foundation of a just society.

    This was a very bold claim, and had Rawls been able to fulfill its promise, it would have been a monumental achievement. As we shall see, Rawls very early recognized that the original version of the theorem was unprovable, and indeed false. In response to this realization, he made sweeping changes to his theory, resulting in the distinctive form that the theory takes in A Theory of Justice, but unfortunately, the revised theory is not more defensible than the original. Rawls himself seems to have realized this fact, for while repeating the language of "theorem" and "proof," he very considerably backs away from the strong claims that he made in the earliest published version of his theory.

    Before we begin the detailed examination of the argument, let me take just a moment to explain why I believe it is appropriate to bring the tools and insights of Game Theory to bear on A Theory of Justice. That is, after all, not the customary manner in which we engage with the arguments of Hobbes, Locke, Rousseau, Mill, Kant, or any of the other great figures of the Western tradition of democratic political theory. Quite simply, the reason is this: A great part of the plausibility of Rawls' theses derives from his claim that they can be grounded in a formal argument of Bargaining Theory. Absent that claim, the reader is left simply to contemplate Rawls' political theory and consider whether he or she likes it. The argument for the theory is, when all is said and done, the claim that the two principles would be chosen by rationally self-interested individuals situated in what Rawls eventually came to call the Original Position. If that is simply not true, then it is hard to see what other justification Rawls has for his theory.

    It is actually rather difficult to figure out exactly what Rawls' Two Principles mean, and the only way I can see to grapple with them is to take Rawls at his word that they are the solution to a bargaining game, and then see how we might so construe them. In this case, as we shall see, the formal machinery of Game Theory is quite helpful in guiding us to turn Rawls' non-technical language into something precise enough to be subjected to analysis.

    It will be useful for our purposes to begin with the earliest statement of Rawls' theory, as it appeared in an article entitled "Justice as Fairness," published in 1962 in an important collective volume of essays called Philosophy, Politics, and Society, Second Series, edited by Peter Laslett and W. G. Runciman. Two passages from the essay will set things up for the first stage of my analysis.

    "The conception of justice which I want to develop," Rawls writes, "may be stated in the form of two principles as follows: first, each person participating in a practice, or affected by it, has an equal right to the most extensive liberty compatible with a like liberty for all; and second, inequalities are arbitrary unless it is reasonable to expect that they will work out for everyone's advantage, and provided the positions and offices to which they attach, or from which they may be gained, are open to all."

    These principles, Rawls says, would be agreed upon unanimously in a deliberation that he characterizes roughly in the way that "state of nature" political theorists describe the agreement on the Social Contract that constitutes a nation. Although he acknowledges that his remarks "are not offered as a rigorous proof" that persons engaged in this deliberation would agree on the two principles, such a proof requiring "a more elaborate and formal argument," nevertheless, he goes on to say:

    "[T]he proposition I seek to establish is a necessary one, that is, it is intended as a theorem: namely, that when mutually self-interested and rational persons confront one another in typical circumstances of justice, and when they are required by a procedure expressing the constraints of having a morality to jointly acknowledge principles by which their claims on the design of their common practice are to be judged, they will settle on these two principles as restrictions governing the assignment of rights and duties, and thereby accept them as limiting their rights against one another."

    I think it is patently clear that Rawls in these passages is laying claim to the rigor and demonstrative power of Game Theory, at least in its somewhat looser form as Bargaining Theory. He seeks to show that his proposition is a necessary one; he asserts that it is intended as a theorem. Throughout the long evolution and transformation of his theory, Rawls never gave up this claim, for all that he also never came close to providing an argument for it. It is, I believe, the heart and soul of his entire enterprise. Without it, he has nothing but a rather affecting, albeit extremely murky, expression of his personal preferences in social organization.

    What can Game Theory tell us about Rawls' claim? There are two questions that we must try to answer: What do his two principles mean? and Is his assertion a theorem that can be proved with necessity?

    First, a problem: In the original statement of the principles, Rawls states his first principle thus: "each person participating in a practice, or affected by it, has an equal right to the most extensive liberty compatible with a like liberty for all. " In A Theory of Justice, however, the reference to practices is omitted, and instead we get "Each person is to have an equal right to the most extensive basic liberty compatible with a similar liberty for others." [p. 60] Eventually, this is tweaked a bit, and becomes "Each person is to have an equal right to the most extensive total system of equal basic liberties compatible with a similar system of liberty for all." [p. 302] The original formulation is thus intended to apply to practices, such as marriage, or capitalism, or the military, or the judicial system. The final formulation applies to nothing less than the total organization of a society. The shift, as we shall see, makes it forbiddingly difficult to figure out what the principle actually means.

    The first principle, in all of its variants, uses the phrase "the most extensive." That implies that one can rank alternative arrangements of a practice, or, alternative sets of fundamental or constitutional arrangements, in order of the degree of liberty that they embody or promise or make possible or guarantee. But as the term "liberty" is ordinarily used in the context of debates about political systems, it refers to a wide variety of institutional arrangements or guarantees that vary along multiple dimensions. The right to trial by a jury of one's peers, we may suppose, is a form of liberty. So is the right of all adults to vote periodically in elections to select the members of the government. Is a system of government with the first but not the second a more extensive or a less extensive liberty than a system of government with the second but not the first? One might reply, it does not matter, because a system of government with both is superior to either. Hence it would be Pareto preferred to either. But suppose there are liberties that in some of their forms are incompatible, in the sense that guaranteeing one interferes with guaranteeing the other.

    For example [if I may be a trifle facetious], imagine a society consisting solely of authors and literary critics [these days, what with the internet, it does seem as though everyone is either an author or a critic, and sometimes both]. Now authors wish to be free of what they consider unfounded attacks on their writings, and critics wish to be free of what they consider unjustified limitations on their critiques. So the critics will prefer the American system of libel law, which gives very broad latitude to critics, and authors will prefer the British system, which favors those supposedly libeled [unless, of course, the authors wish, in their writings, to say nasty things about the critics, in which case the situation is more complex.] There is no way in this situation simultaneously to maximize the liberty of both groups, since each extension of the liberty of one will be experienced as a loss of liberty by the other. Consequently, as the authors and critics gather to bargain on the founding principles of liberty in their society, they will find it impossible to achieve unanimity on Rawls' first principle, because every attempt to spell out what it means will have them at loggerheads. This problem is not at all trivial, for all that my example may make it seem so. It is referred to in economics as the Indexing Problem, and it will crop again in a different guise in the final version of Rawls' theory.